Prove by Mean Value Theorem $frac{x}{1+x}<ln(1+x)0$











up vote
3
down vote

favorite
1












Prove for $x>0$
$$
frac{x}{1+x}<ln(1+x)<x
$$
I tried writing $ln(1+x)=ln(1+x)-ln(1)$ and using the MVT for the $(1,1+x)$ interval. I eventually could prove the inequality but how do I have to prove even for $(0,1)$










share|cite|improve this question
























  • can it be done without Taylor?
    – user224677
    Mar 18 '15 at 18:08










  • I know but we haven't quite studied Taylor series so if you could help with some hint, I just don't know how to prove for (0;1) interval because the rest I have done
    – user224677
    Mar 18 '15 at 18:11










  • I will think on it; I don't currently have it solved either :)
    – graydad
    Mar 18 '15 at 18:13















up vote
3
down vote

favorite
1












Prove for $x>0$
$$
frac{x}{1+x}<ln(1+x)<x
$$
I tried writing $ln(1+x)=ln(1+x)-ln(1)$ and using the MVT for the $(1,1+x)$ interval. I eventually could prove the inequality but how do I have to prove even for $(0,1)$










share|cite|improve this question
























  • can it be done without Taylor?
    – user224677
    Mar 18 '15 at 18:08










  • I know but we haven't quite studied Taylor series so if you could help with some hint, I just don't know how to prove for (0;1) interval because the rest I have done
    – user224677
    Mar 18 '15 at 18:11










  • I will think on it; I don't currently have it solved either :)
    – graydad
    Mar 18 '15 at 18:13













up vote
3
down vote

favorite
1









up vote
3
down vote

favorite
1






1





Prove for $x>0$
$$
frac{x}{1+x}<ln(1+x)<x
$$
I tried writing $ln(1+x)=ln(1+x)-ln(1)$ and using the MVT for the $(1,1+x)$ interval. I eventually could prove the inequality but how do I have to prove even for $(0,1)$










share|cite|improve this question















Prove for $x>0$
$$
frac{x}{1+x}<ln(1+x)<x
$$
I tried writing $ln(1+x)=ln(1+x)-ln(1)$ and using the MVT for the $(1,1+x)$ interval. I eventually could prove the inequality but how do I have to prove even for $(0,1)$







calculus inequality logarithms






share|cite|improve this question















share|cite|improve this question













share|cite|improve this question




share|cite|improve this question








edited Nov 18 at 6:35









Martin Sleziak

44.4k7115268




44.4k7115268










asked Mar 18 '15 at 17:55









user224677

161




161












  • can it be done without Taylor?
    – user224677
    Mar 18 '15 at 18:08










  • I know but we haven't quite studied Taylor series so if you could help with some hint, I just don't know how to prove for (0;1) interval because the rest I have done
    – user224677
    Mar 18 '15 at 18:11










  • I will think on it; I don't currently have it solved either :)
    – graydad
    Mar 18 '15 at 18:13


















  • can it be done without Taylor?
    – user224677
    Mar 18 '15 at 18:08










  • I know but we haven't quite studied Taylor series so if you could help with some hint, I just don't know how to prove for (0;1) interval because the rest I have done
    – user224677
    Mar 18 '15 at 18:11










  • I will think on it; I don't currently have it solved either :)
    – graydad
    Mar 18 '15 at 18:13
















can it be done without Taylor?
– user224677
Mar 18 '15 at 18:08




can it be done without Taylor?
– user224677
Mar 18 '15 at 18:08












I know but we haven't quite studied Taylor series so if you could help with some hint, I just don't know how to prove for (0;1) interval because the rest I have done
– user224677
Mar 18 '15 at 18:11




I know but we haven't quite studied Taylor series so if you could help with some hint, I just don't know how to prove for (0;1) interval because the rest I have done
– user224677
Mar 18 '15 at 18:11












I will think on it; I don't currently have it solved either :)
– graydad
Mar 18 '15 at 18:13




I will think on it; I don't currently have it solved either :)
– graydad
Mar 18 '15 at 18:13










1 Answer
1






active

oldest

votes

















up vote
2
down vote













By the mean value theorem, given $x > 0$, there exists $c in (0,x)$ such that $f(x) - f(0) = f'(c)x$, i.e., $ln(1 + x) = frac{x}{1 + c}$. Since $0 < c < x$, $frac{1}{1 + x} < frac{1}{1 + c} < 1$. Therefore $frac{x}{1 + x} < frac{x}{1 + c} < x$, i.e.,



$$frac{x}{1 + x} < ln(1 + x) < x.$$






share|cite|improve this answer





















    Your Answer





    StackExchange.ifUsing("editor", function () {
    return StackExchange.using("mathjaxEditing", function () {
    StackExchange.MarkdownEditor.creationCallbacks.add(function (editor, postfix) {
    StackExchange.mathjaxEditing.prepareWmdForMathJax(editor, postfix, [["$", "$"], ["\\(","\\)"]]);
    });
    });
    }, "mathjax-editing");

    StackExchange.ready(function() {
    var channelOptions = {
    tags: "".split(" "),
    id: "69"
    };
    initTagRenderer("".split(" "), "".split(" "), channelOptions);

    StackExchange.using("externalEditor", function() {
    // Have to fire editor after snippets, if snippets enabled
    if (StackExchange.settings.snippets.snippetsEnabled) {
    StackExchange.using("snippets", function() {
    createEditor();
    });
    }
    else {
    createEditor();
    }
    });

    function createEditor() {
    StackExchange.prepareEditor({
    heartbeatType: 'answer',
    convertImagesToLinks: true,
    noModals: true,
    showLowRepImageUploadWarning: true,
    reputationToPostImages: 10,
    bindNavPrevention: true,
    postfix: "",
    imageUploader: {
    brandingHtml: "Powered by u003ca class="icon-imgur-white" href="https://imgur.com/"u003eu003c/au003e",
    contentPolicyHtml: "User contributions licensed under u003ca href="https://creativecommons.org/licenses/by-sa/3.0/"u003ecc by-sa 3.0 with attribution requiredu003c/au003e u003ca href="https://stackoverflow.com/legal/content-policy"u003e(content policy)u003c/au003e",
    allowUrls: true
    },
    noCode: true, onDemand: true,
    discardSelector: ".discard-answer"
    ,immediatelyShowMarkdownHelp:true
    });


    }
    });














     

    draft saved


    draft discarded


















    StackExchange.ready(
    function () {
    StackExchange.openid.initPostLogin('.new-post-login', 'https%3a%2f%2fmath.stackexchange.com%2fquestions%2f1195842%2fprove-by-mean-value-theorem-fracx1x-ln1xx-for-x0%23new-answer', 'question_page');
    }
    );

    Post as a guest















    Required, but never shown

























    1 Answer
    1






    active

    oldest

    votes








    1 Answer
    1






    active

    oldest

    votes









    active

    oldest

    votes






    active

    oldest

    votes








    up vote
    2
    down vote













    By the mean value theorem, given $x > 0$, there exists $c in (0,x)$ such that $f(x) - f(0) = f'(c)x$, i.e., $ln(1 + x) = frac{x}{1 + c}$. Since $0 < c < x$, $frac{1}{1 + x} < frac{1}{1 + c} < 1$. Therefore $frac{x}{1 + x} < frac{x}{1 + c} < x$, i.e.,



    $$frac{x}{1 + x} < ln(1 + x) < x.$$






    share|cite|improve this answer

























      up vote
      2
      down vote













      By the mean value theorem, given $x > 0$, there exists $c in (0,x)$ such that $f(x) - f(0) = f'(c)x$, i.e., $ln(1 + x) = frac{x}{1 + c}$. Since $0 < c < x$, $frac{1}{1 + x} < frac{1}{1 + c} < 1$. Therefore $frac{x}{1 + x} < frac{x}{1 + c} < x$, i.e.,



      $$frac{x}{1 + x} < ln(1 + x) < x.$$






      share|cite|improve this answer























        up vote
        2
        down vote










        up vote
        2
        down vote









        By the mean value theorem, given $x > 0$, there exists $c in (0,x)$ such that $f(x) - f(0) = f'(c)x$, i.e., $ln(1 + x) = frac{x}{1 + c}$. Since $0 < c < x$, $frac{1}{1 + x} < frac{1}{1 + c} < 1$. Therefore $frac{x}{1 + x} < frac{x}{1 + c} < x$, i.e.,



        $$frac{x}{1 + x} < ln(1 + x) < x.$$






        share|cite|improve this answer












        By the mean value theorem, given $x > 0$, there exists $c in (0,x)$ such that $f(x) - f(0) = f'(c)x$, i.e., $ln(1 + x) = frac{x}{1 + c}$. Since $0 < c < x$, $frac{1}{1 + x} < frac{1}{1 + c} < 1$. Therefore $frac{x}{1 + x} < frac{x}{1 + c} < x$, i.e.,



        $$frac{x}{1 + x} < ln(1 + x) < x.$$







        share|cite|improve this answer












        share|cite|improve this answer



        share|cite|improve this answer










        answered Mar 18 '15 at 18:15









        kobe

        34.5k22247




        34.5k22247






























             

            draft saved


            draft discarded



















































             


            draft saved


            draft discarded














            StackExchange.ready(
            function () {
            StackExchange.openid.initPostLogin('.new-post-login', 'https%3a%2f%2fmath.stackexchange.com%2fquestions%2f1195842%2fprove-by-mean-value-theorem-fracx1x-ln1xx-for-x0%23new-answer', 'question_page');
            }
            );

            Post as a guest















            Required, but never shown





















































            Required, but never shown














            Required, but never shown












            Required, but never shown







            Required, but never shown

































            Required, but never shown














            Required, but never shown












            Required, but never shown







            Required, but never shown







            Popular posts from this blog

            Quarter-circle Tiles

            build a pushdown automaton that recognizes the reverse language of a given pushdown automaton?

            Mont Emei